- PowerScore Staff
- Posts: 5972
- Joined: Mar 25, 2011
- Thu Oct 29, 2020 5:43 pm
#80544
This game is also discussed in our Podcast: LSAT Podcast Episode 70: The May 2020 LSAT-Flex Logic Games Section
Complete Question Explanation
(The complete setup for this game can be found here: viewtopic.php?t=33052)
The correct answer choice is (A).
The question stem places L on day 7 and O on day 8. L on day 7 immediately triggers the second rule, placing L on day 2 as well:
To make hypotheticals here, don't just randomly start plugging in variables to spaces. Instead, focus on the areas of greatest effect. The first rule still needs requirement, and since it takes up two spaces (1-5) AND has Not Laws (H), it's a great starting point. Only three options exist for the 1-5 spaces: G, J, and M. Since G is a random, start with J and M first (side note: come back to this point later and consider why we bypassed G for now—as a random it is very weak; J and M are far more powerful, and thus better to start with for hypotheticals):
J is performed on days 1-5
With J in 1-5, it's immediately apparent that only days 3-4 are available for the HM block. That then forces the last variable, G, into day 6.
M is performed on days 1-5
With M in 1-5, the only possible position of H to satisfy the fourth rule is on day 4. that leaves J and G to be placed. Because J must be performed before the first instance of H per the last rule, that forces J to be performed on day 3. G is then left to perform on day 6.
Answer choice (A) is now the only remaining answer that has not been eliminated, and as further confirmation, both of our hypotheticals conform to answer choice (A)
Answer choice (A): This is the correct answer choice, and is proven correct by process of elimination as shown above.
Answer choice (B): While this answer choice could be true, it does not have to be true, and is thus incorrect.
Answer choice (C): While this answer choice could be true, it does not have to be true, and is thus incorrect.
Answer choice (D): While this answer choice could be true, it does not have to be true, and is thus incorrect.
Answer choice (E): While this answer choice could be true, it does not have to be true, and is thus incorrect.
Complete Question Explanation
(The complete setup for this game can be found here: viewtopic.php?t=33052)
The correct answer choice is (A).
The question stem places L on day 7 and O on day 8. L on day 7 immediately triggers the second rule, placing L on day 2 as well:
- ___ _L_ ___ ___ ___ ___ _L_ _O_
1 2 3 4 5 6 7 8
To make hypotheticals here, don't just randomly start plugging in variables to spaces. Instead, focus on the areas of greatest effect. The first rule still needs requirement, and since it takes up two spaces (1-5) AND has Not Laws (H), it's a great starting point. Only three options exist for the 1-5 spaces: G, J, and M. Since G is a random, start with J and M first (side note: come back to this point later and consider why we bypassed G for now—as a random it is very weak; J and M are far more powerful, and thus better to start with for hypotheticals):
J is performed on days 1-5
With J in 1-5, it's immediately apparent that only days 3-4 are available for the HM block. That then forces the last variable, G, into day 6.
- _J_ _L_ _H_ _M_ _J_ _G_ _L_ _O_
1 2 3 4 5 6 7 8
M is performed on days 1-5
With M in 1-5, the only possible position of H to satisfy the fourth rule is on day 4. that leaves J and G to be placed. Because J must be performed before the first instance of H per the last rule, that forces J to be performed on day 3. G is then left to perform on day 6.
- _M_ _L_ _J_ _H_ _M_ _G_ _L_ _O_
1 2 3 4 5 6 7 8
Answer choice (A) is now the only remaining answer that has not been eliminated, and as further confirmation, both of our hypotheticals conform to answer choice (A)
Answer choice (A): This is the correct answer choice, and is proven correct by process of elimination as shown above.
Answer choice (B): While this answer choice could be true, it does not have to be true, and is thus incorrect.
Answer choice (C): While this answer choice could be true, it does not have to be true, and is thus incorrect.
Answer choice (D): While this answer choice could be true, it does not have to be true, and is thus incorrect.
Answer choice (E): While this answer choice could be true, it does not have to be true, and is thus incorrect.
Dave Killoran
PowerScore Test Preparation
Follow me on X/Twitter at http://twitter.com/DaveKilloran
My LSAT Articles: http://blog.powerscore.com/lsat/author/dave-killoran
PowerScore Podcast: http://www.powerscore.com/lsat/podcast/
PowerScore Test Preparation
Follow me on X/Twitter at http://twitter.com/DaveKilloran
My LSAT Articles: http://blog.powerscore.com/lsat/author/dave-killoran
PowerScore Podcast: http://www.powerscore.com/lsat/podcast/